Saturday 28 September 2019

UPSC 2019 WEEKLY CURRENT AFFAIRS



 TECH PKPRAJAPAT

WEEKLY CURRENT AFF TEST 22 TO 28 SEPT. 2019



1. Consider the following statements with respect to ‘Cyberdome Project’
  1. It makes a collective coordination among the Government departments and agencies, academia, research groups, non-profitable organizations, individual experts from the community, ethical hackers, private organizations, and other law enforcement agencies in the country with an aim of providing a safe and secure cyber world for each and every citizen in the state.
  2. It is jointly launched by Ministry of Home affair and NITI Aayog to replace the CCTNS and NATGRID.

Select the correct statements

  1. 1 Only
  2. 2 Only
  3. Both 1 and 2
  4. Neither 1 nor 2

2. Consider the following statements with respect to ‘CARICOM’

  1. It is an organisation of Caribbean nations and dependencies having primary objectives to promote economic integration and cooperation among its members.
  2. All of the associate members of CARICOM are British overseas territories.

Select the correct statements

  1. 1 Only
  2. 2 Only
  3. Both 1 and 2
  4. Neither 1 nor 2

3. ‘Abqaiq’ was in news recently. Where is it located?
  1. Saudi Arabia
  2. Iran
  3. Syria
  4. Lebanon

4. ‘1.5-Degree Report (SR 1.5)’ is published by 
  1. The Intergovernmental Panel on Climate Change
  2. The UNEP Secretariat
  3. The UNFCCC Secretariat
  4. The World Meteorological Organization

5. ‘Shondol dance’ was in news recently. It is associated 
  1. Rayalaseema
  2. Marathwada
  3. Ladakh
  4. Saurashtra

6. Consider the following statements with respect to ‘Naropa Festival’
  1. It takes places every 12 years.
  2. It celebrates the life of Jain philosopher and scholar Naropa.
  3.  
Select the correct statements
  1. 1 Only
  2. 2 Only
  3. Both 1 and 2
  4. Neither 1 nor 2


7. The Genetic Engineering Appraisal Committee (GEAC) is constituted under the<
  1. Food Safety and Standards Act, 2006
  2. Geographical Indications of Goods (Registration and Protection) Act, 1999
  3. Environment (Protection) Act, 1986
  4. Wildlife (Protection) Act, 1972

8. Consider the following statements with respect to ‘Participatory Guarantee Scheme (PGS)’

  1. It is a process of certifying organic products, which ensures that their production takes place in accordance with laid-down quality standards.
  2. Individual farmers or group of farmers smaller than five members are not covered under PGS.
Select the correct statements

  1. 1 Only
  2. 2 Only
  3. Both 1 and 2
  4. Neither 1 nor 2

9 Consider the following statements with respect to ‘PACEsetter Fund’

  1. The mission of the PACEsetter Fund is to accelerate the commercialization of innovative off-grid clean energy access solutions by providing early-stage grant funding that would allow businesses to develop and test innovative products, business models and systems.
  2. It was constituted by India and the USA in 2015 as a joint fund.

Select the correct statements 

  1. 1 Only
  2. 2 Only
  3. Both 1 and 2
  4. Neither 1 nor 2

10. Consider the following statements with respect to ‘GIFCT’

  1. It is established by International Telecommunication Union.
  2. It is dedicated to disrupting terrorist abuse of members’ digital platforms.

Select the correct statements 

  1. 1 Only
  2. 2 Only
  3. Both 1 and 2
  4. Neither 1 nor 2

11. A new Leadership Group for Industry Transition that will drive transformation in hard-to-decarbonize and energy-intensive sectors was announced at the UN Climate Action Summit. Consider the following statements in this regard.

  1. It will be supported by the World Economic Forum.
  2. It was initiated by the United States of America.

Select the correct statements

  1. 1 Only
  2. 2 Only
  3. Both 1 and 2
  4. Neither 1 nor 2

12. Which of the following statements with respect to ‘CPGRAMS’ is true?

  1. It is a single window system for Candidates and Political Parties to apply for permissions
  2. It is a Vehicle Management System with the facility of Issuance of requisition letters for vehicles
  3. It is a Public Grievance Redressal System with respect to ‘Right to Information (RTI)’ matters
  4. None of the above

13. Consider the following statements with respect to ‘Global Goalkeeper Award’
  1. The award celebrates a political leader who has “demonstrated their commitment to the Global Goals through impactful work in their country and/or globally.
  2. It is a “special recognition” by the Bill and Melinda Gates Foundation.

Select the correct statements

  1. 1 Only
  2. 2 Only
  3. Both 1 and 2
  4. Neither 1 nor 2

14. Which of the following statements is/are correct?

  1. Methane can be synthesised with water and carbon dioxide in space.
  2. Methane burns on its own upon coming in contact with oxygen.

Select the correct statements
  1. 1 Only
  2. 2 Only
  3. Both 1 and 2
  4. Neither 1 nor 2

15. Consider the following statements regarding Financial Action Task Force (FATF).

1. The Financial Action Task Force (FATF) is an inter-governmental body established on the
initiative of the G20.

2. FATF works to generate the necessary political will to bring about national legislative and regulatory reforms for combating money laundering and terrorist financing.
3. A country is blacklisted if it is non-cooperative in the global fight against money laundering and terrorist financing.
4. The FATF has developed a series of Recommendations that are recognised as the international standard for combating of money laundering and the financing of terrorism and proliferation of weapons of mass destruction.

Which of the above statements is/are correct?

 a) 1, 2, 3
 b) 1, 3, 4
 c) 2, 3, 4
 d) 1, 2, 3, 4

16. Consider the following statements regarding UN Security Council.

1. Each year, the General Assembly elects five non-permanent members out of a total of 10, for a two-year term.
2. India has been a non-permanent member of the Security Council seven times previously with the most recent being the 2011–12 term.
3. India’s candidature for a non-permanent seat in the Security Council for the 2021-22 term has been supported by Pakistan and China.

Which of the above statements is/are correct?

 a) 1, 2
 b) 2, 3
 c) 1, 3
 d) 1, 2, 3


17. Consider the following statements regarding Total Polar Compounds (TPC).

1. TPC is used to measure the quality of cooking oil.
2. The level of TPC increases every time oil is re-heated.
3. According to FSSAI regulations, the maximum permissible limits for Total Polar Compounds (TPC) have been set at 50%, beyond which the cooking oil is unsafe for consumption.

Which of the above statements is/are correct?

 a) 1, 2
 b) 1, 3
 c) 2, 3
 d) 1, 2, 3

18. Consider the following statements about Soil-Transmitted Helminths (STH).

1. STH infectionscan lead to malnutrition, impaired mental and physical & cognitive development.
2. STH Infections cannot be prevented by Wearing slippers and shoes.
3. National Deworming Day is observed to deworm all preschool and school-age children between the ages of 1-19 years through schools and Anganwadi Centers.

Which of the above statements is/are correct?

 a) 1, 2
 b) 2, 3
 c) 1, 3
 d) 1, 2, 3

19. Consider the following statements regarding Advisory Board for Banking Frauds (ABBF).

1. Advisory Board for Banking Frauds (ABBF) is been constituted by the RBI to examine bank fraud of over 50 crore and recommend action.
2. It would function as the first level of examination of all large fraud cases before references are made to the investigative agencies.
3. It would also give inputs for policy formulation related to the fraud to the RBI.

Which of the above statements is/are correct?

 a) 1, 2
 b) 1, 3
 c) 2, 3
 d) 1, 2, 3

20. Consider the following statements regarding Vikram Sarabhai.

1. Union Government recently announced “Vikram Sarabhai Journalism Award in Space Science, Technology and Research” to reward journalists who have actively contributed towards the field of space science, applications, and research.
2. Vikram Sarabhai is known as the father of India’s missile programme.
3. Vikram Sarabhai was responsible for bringing cable television to India.

Which of the above statements is/are incorrect?

 a) 1, 2
 b) 1, 3
 c) 2 only
 d) 2, 3

21. Consider the following statements regarding stubble burning.

1. There is an increase in crop residue burning in North India in 2018 compared with that in 2016.
2. Stubble burning results in emission of carbon dioxide, sulphur dioxide, nitrogen dioxide along with particulate matter.
3. Stubble burning does not kill weeds that are resistant to herbicide.

Which of the above statements is/are correct?

 a) 1, 2
 b) 2 only
 c) 2, 3
 d) 1, 2, 3

22. Consider the following statements about Global Environment Outlook (GEO) Report.

1. It is referred to as UN Environment’s flagship environmental assessment, released annually.
2. It provides an integrated analysis of social, economic and environmental trends that have shaped the environment.

Which of the above statements is/are correct?

 a) 1 only
 b) 2 only
 c) Both
 d) None

23. Consider the following statements regarding G20 Forum and G20 Summit.

1. 2019 G20 Summit was the first G20 summit to be hosted by Japan.
2. Pakistan is not a member of G20 Forum.
3. G20 summits focus only on macroeconomy and trade.
4. India has hosted G20 summits twice since its inception.

Which of the above statements is/are incorrect?

 a) 1, 2, 4
 b) 3, 4
 c) 2, 3, 4
 d) 1, 3, 4

24. Consider the following statements.

1. Bureau of Indian Standards is the national agency that is responsible for carrying out certification for instruments and equipments for monitoring emissions and ambient air.
2. National Clean Air Programme is a five-year action plan with an aim of 20-30% reduction of PM2.5 and PM10 concentration by 2024.
3. National Clean Air Programme also includes increasing number of monitoring stations in the country including rural monitoring stations.

Which of the above statements is/are correct?

 a) 1, 2
 b) 1, 3
 c) 2, 3
 d) 1, 2, 3

25. Which of the following are the possible implications when a country adopts negative rate policy.

1. Increases borrowing costs.
2. Help weaken a country’s currency rate by making it a less attractive investment than that of other currencies.
3. Boosts Inflation

Select the correct answer code:

 a) 1, 2
 b) 2 only
 c) 2, 3
 d) 1, 3

26.  ‘Global Assessment of Forest Biodiversity’ report has been released by

 a) International Union for Conservation of Nature
 b) UN Environment
 c) World Wide Fund for Nature
 d) Both b and c

27. Researchers from Indian Institute of Technology Madras (IITM) developed “Shakti” is related to

 a) Climate vulnerability maps developed for Himalayan states
 b) Utilising black carbon soot for treating industrial waste
 c) Microprocessor
 d) Magnetic Graphene

28. Consider the following statements regarding C-Sat-Fi technology.

1. C-Sat-Fi technology is based on satellite communication to extend connectivity to the unserved areas including the remote islands and difficult terrains.
2. C-Sat-Fi can work on any WiFi enabled phone.

Which of the above statements is/are correct?

 a) 1 only
 b) 2 only
 c) Both
 d) None

29. Consider the following statements regarding Special Data Dissemination Standard (SDDS).

1. Special Data Dissemination Standard (SDDS) is the World Bank standard to guide member countries in the dissemination of national statistics to the public.
2. According to “Annual Observance Report of the Special Data Dissemination Standard for 2018”, India is in the top position among Asian countries to comply with multiple requirements prescribed in the Special Data Dissemination Standard (SDDS).

Which of the above statements is/are correct?

 a) 1 only
 b) 2 only
 c) Both
 d) None

30. Consider the following statements regarding Notifiable Disease.

1. A notifiable disease is any disease that is required by law to be reported to government authorities.
2. The onus of notifying any disease and the implementation lies with the state government.
3. Any failure to report a notifiable disease is a criminal offence and the state government can take necessary actions against defaulters.

Which of the above statements is/are correct?

 a) 1, 2
 b) 1, 3
 c) 2, 3
 d) 1, 2, 3

31. Consider the following statements regarding Know India Programme.

Know India Programme is a flagship programme of Ministry of External Affairsfor engagement with Indian origin youth to enhance their awareness about India and its cultural heritage.
Any Indian origin youth between 18-30 years is eligible for participating in KIP.
Those who have not visited India before will be given preference.
Which of the above statements is/are correct?

 a) 1, 2
 b) 1, 3
 c) 2, 3
 d) 1, 2, 3

32. Consider the following statements about Consumer Protection Act, 1986

1. It makes provision for the establishment of consumer councils and other authorities for the settlement of consumers’ disputes.

2. Wherever possible it assures access to goods and services at competitive prices.

3. It protects the consumers against marketing of hazardous goods and services.

Seek redressal on account of unscrupulous exploitation of consumers.
Which of the above statements is/are correct?

 a) 1, 2
 b) 1, 2, 4
 c) 1, 3, 4
 d) 1, 2, 3, 4

33. Mitra crater, recently seen in news is present on

 a) Jupiter
 b) Mars
 c) Titan
 d) None of the above

34. With reference to National Investigation Agency:

  1. It deals with offenses of terrorism only
  2. it does not extend to citizens of India outside India
  3. It extends to persons on ships and aircrafts registered in India wherever they may be

Which of above statements is/ are incorrect?

a) 1 only
b) 1 and 2 only
c) 1, 2 and 3
d) None

35. Yellow Vest recently seen in news was a protest in which country?

a) United Kingdom
b) USA
c) France
d) Germany

36. Consider the following statements about IMMSAREX

  1. It is the world’s largest international maritime warfare exercise
  2. It is hosted and administered by the United States Navy

Which of above statements is/ are correct?

a) 1 only
b) 2 only
c) Both
d) None


37. Consider the following statements:
  1. “Khan Quest” is a bilateral exercise between India and Mongolia.
  2. It is an annual exercise organised for sharing of practices for multinational peacekeeping operations.

Which of the given statement/s is/are NOT correct?

a. 1 only
b. 2 only
c.Both 1 and 2
d. Neither 1 nor 2

38. Consider the following statements:

  1. Fiscal Deficit is the sum of budgetary deficit and net increase in internal and external borrowings.
  2. It is an indication of the total borrowings needed by the government.
  3. Fiscal responsibility and Budget Management Act concerns both Fiscal and Revenue deficit.

Which of the given statement/s is/are correct?

a. 1 only
b. 1 and 3 only
c. 2 and 3 only
d. 1, 2 and 3

Q39. Consider the following statements:
  1. GST Council is a statutory body for making recommendations to the Union and State Government on issues related to Goods and Service Tax.
  2. The GST Council will suo moto notify the rate of tax to be levied under the Central Goods and Services Tax Act.

Which of the given statement/s is/are NOT correct?

a. 1 only
b. 2 only
c. Both 1 and 2
d. Neither 1 nor 2

40. Consider the following statements with respect to Biomining:
  1. It is the process of using microorganisms to extract metals from rock ores or mine waste.
  2. Bioleacing, biooxidation and bioremediation are biomining processes.

Which of the given statement/s is/are correct?

a. 1 only
b. 2 only
c. Both 1 and 2
d. Neither 1 nor 2

41. Consider the following statements with respect to The Framework Convention on
Tobacco Control (FCTC):

  1. It is the world’s first public health treaty enacted under the World Health Organization (WHO).
  2. India has ratified FCTC.
  3. The treaty is legally binding on the parties to the convention.

Which of the given statement/s is/are correct?

a. 1 and 2 only
b. 2 only
c. 1 and 3 only
d. 1, 2 and 3

42. “Bhut Jolokia” recently seen in news is:

a. A tribal dance performed by the lotha tribesmen.
b. The Naga Chilli which is one of the hottest chilies in the world.
c. India’s first dragon blood oozing tree.
d. An instrument made out of dried gourd and a thin rubber drum associated with Bengali folk music.

43. Consider the following statements:

  1. Aadhaar Cards are issued by Unique Identification Authority of India (UIDAI)  only.
  2. UIDAI functions under NITI Aayog.
  3. Foreign Nationals residing in India are eligible to apply for Aadhaar Card.

Which of the given statement/s is/are correct?

a. 1 only
b. 1 and 2 only
c. 2 and 3 only
d. 1 and 3 only

44. Which of the following are classified as Critical Vulnerable Coastal Areas?

  1. Sundarban region of West Bengal
  2. Vembanad in Kerala
  3. Bhitarkanika in Odisha

Choose the correct option:

a. 1 only
b. 1 and 2 only
c. 1, 2 and 3
d. 3 only

Q45. Consider the following statements:

  1. The Sustainable Development Goal 3 dealing with consolidated goal on health explicitly mentions Tuberculosis.
  2. The Government is committed to achieving the target of TB elimination by 2030 in line with the WHO targets for TB elimination.

Which of the given statement/s is/are correct?

a. 1 only
b. 2 only
c. Both 1 and 2
d. Neither 1 nor 2

Q46.SITMEX is a trilateral exercise between which of the following countries:

 ( a ) Singapore-Indonesia-Thailand
 ( b ) Singapore-India-Thailand
 ( c ) Sri Lanka-Indonesia-Thailand
 ( d ) Sri Lanka-India-Thailand

Q47.NATGRID is a robust intelligence gathering mechanism related to banking, immigration, individual tax payers, air and train travels.

Which of the following agencies will be able to ac

1.Enforcement Directorate (ED)
2.Directorate of Revenue Intelligence (DRI)
3.Financial Intelligence Unit (FIU)
4. Central Board of Direct Taxes (CBDT)
5. Central Board of Excise and Customs (CBEC)

cess NATGRID data on real time basis?

 ( a ) 1 2 and 3
 ( b ) 2 3 4 & 5
 ( c ) 1 2 4 & 5
 ( d ) All the above

Q48.Consider the following statements regarding the Ayushman Bharat-National Health Protection Scheme

1) It provides a coverage up to 5 lakh rupees per family per year for secondary and tertiary care hospitalization.
2) Benefits of the scheme are portable in empaneled hospitals across the country

Which of the statements above are true?

 (a) 1 only
 (b) 2 only
 (c) Both 1 and 2
 (d) Neither 1 nor 2

49.Consider the following statements regarding ‘cleanliness survey’ in India

1)Swachh Sarvekshan, an annual cleanliness survey under the Swachh Bharat Abhiyan is carried out by Quality Council of India
2)Indore was ranked as the cleanest city on the cleanliness index of the Ministry of Urban Development in 2018.

Which of the statements above are true?

 ( a ) 1 only
 ( b ) 2 only
 ( c ) Both 1 and 2
 ( d ) Neither 1 nor 2

50. Consider the following statements regarding the Kisan Credit Card scheme

1)Tenant farmers, oral lessees and share croppers are not eligible for benefits under this scheme
2)Beneficiaries can avail personal accident insurance coverage under this scheme
3)Crop loans under this scheme are provided only for selected crops

Choose the correct answer from the options given below

 ( a ) 1 and 2 only
 ( b ) 2 and 3 only
 ( c ) 1 and 3 only
 ( d ) 1 2 and 3

51.Which among the following statements best describes the term interest subvension that is often discussed in the news?

 ( a ) An interest subsidy given by the government on loans for certain important sectors
 ( b ) Interest free loans from the government
 ( c ) Writing-off interests on farm loans
 ( d ) Government mandated priority sector lending at an interest rate lower than market interest rates

Q52.Mattavilasaprahasanam, a Sanskrit play, was composed by which of the following Pallava kings?

 ( a ) Mahendravaraman I
 ( b ) Narasimhavarman I
 ( c ) Nandivarman II
 ( d ) Rajasimha

Q53. Which of the following feature/(s) is/are related to Pallava architecture?

1)Descent of the Ganges
2)Penance of Arjuna
3)Panchapandava Rathas
4)Varaha Madapam
5)Bhitari pillar

Choose the correct option using the codes given below:

 ( a ) 1 3 and 4
 ( b ) 2 4 and 5
 ( c ) 1 2 3 and 4
 ( d ) 1 2 3 4 and 5

Q54.In the context of Chalukya administration, consider the following statements:

1)The Chalukya administration was highly centralized unlike that of the Pallavas and the Cholas.
2)They had great maritime power.
3)The Badami Chalukyas were Brahmanical Hindus who consistently prosecuted Buddhists and Jains.

Select the correct option using the codes given below

 ( a ) 1 and 2 only
 ( b ) 2 and 3 only
 ( c ) 1 and 3 only
 ( d ) 1 2 and 3


55. Consider the following statements about the Unique Methods of Management and treatment of Inherited Disorders (UMMID) initiative to tackle the menace of genetic diseases in newborns.

1. It was launched by Ministry of Science & Technology.
2. It aims to establish NIDAN Kendras to provide counselling, prenatal testing and diagnosis, management, and multidisciplinary care in Government Hospitals.
3. It aims to produce skilled clinicians in Human Genetics.

Which of the above statement(s) is/are correct?

 (a) 2 and 3 only
 (b) 1 and 2 only
 (c) 3 only
 (d) 1 2 and 3

Q56."Tiger Triumph” is a:

 (a ) Latest Tiger Census held in Jim Corbett National Park.
 (b ) New motorcycle launched running on hydrogen cell.
 (c ) First tri-services exercise between India and United States.
 (d ) Victory over Tiger Hill by Indian forces during Kargil War.

Q57.Consider the following statements:

1) Guru Gobind Singh was the tenth and last of the human gurus of Sikhism.
2) He declared that he would create a Panth to challenge the tyrant rulers which led to the institution of the Khalsa Sect.

Which of the above statements is/are correct?

 (a ) 1 only
 (b ) 2 only
 (c ) Both 1 and 2
 (d ) Neither 1 nor 2

Q58. Consider the following statements about the Bahmani Kingdom:

1) The first capital of Bahmani kingdom was Gulbarga.
2) The kingdom disintegrated into five independent sultanates to form the Deccan Sultanates.

Which of the above statements is/are correct?

 (a) 1 only
 (b) 2 only
 (c) Both 1 and 2
 (d) Neither 1 nor 2

59.Which among the following is / are classified as critically endangered in India as per the IUCN Red List of Threatened Species?

1. Gharial
2. One Horned Rhinoceros
3. Bengal Florican
4. Snow Leopard

Select the correct answer using the codes given below:

 (a) 1 and 3 only
 (b) 2 and 4 only
 (c) 1 2 and 3 only
 (d) 1 2 3 4

60.Consider the following wetlands in India:

1. Keoladeo National Park, Rajasthan
2. Loktak Lake, Manipur
3. Chilika Lake, Odisha

 Which among the above are in the Montreux Accord?

 (a) 1 only
 (b) 1 and 2 only
 (c) 2 and 3 only
 (d) 1 2 3

61. Sea turtles occupy a unique position within the food web. Consider the following statements regarding their role:

1. They make coral reefs healthy.

2. They help reverse the flow of nutrients from land to sea.

Which among the above statements is / are correct?

 ( a ) 1 only
 ( b ) 2 only
 ( c ) Both 1 and 2
 ( d ) Neither 1 nor 2

Q62.Consider the following statements regarding the National Pension Scheme

1) Withdrawals from the National pension Scheme is completely tax free.
2) The 2019-20 budget increased the mandatory contribution by the Central Government for employees covered under NPS from the existing 10% of salary to 14%.

Which of the statements given above are true?

 (a) 1 only
 (b) 2 only
 (c) Both 1 and 2
 (d) Neither 1 nor 2

63.The Pradhan Mantri Ujjwala Yojna is connected with which of the following?

 (a) Last mile rural electrification
 (b) Use of solar panels for electrification in rural areas
 (c) Installing solar street lamps in national highways across India
 (d) Providing LPG to economically weaker sections

64. Consider the following statements about the Unique Methods of Management and treatment of Inherited Disorders (UMMID) initiative to tackle the menace of genetic diseases in newborns.

1. It was launched by Ministry of Science & Technology.
2. It aims to establish NIDAN Kendras to provide counselling, prenatal testing and diagnosis, management, and multidisciplinary care in Government Hospitals.
3. It aims to produce skilled clinicians in Human Genetics.

Which of the above statement(s) is/are correct?

 ( a ) 2 and 3 only
 ( b ) 1 and 2 only
 ( c ) 3 only
 ( d ) 1 2 and 3

Q65. "Tiger Triumph” is a:

 ( a ) Latest Tiger Census held in Jim Corbett National Park.
 ( b ) New motorcycle launched running on hydrogen cell.
 ( c ) First tri-services exercise between India and United States.
 ( d ) Victory over Tiger Hill by Indian forces during Kargil War.

66.The Traditional Knowledge Digital Library was established by which of the following agencies in collaboration with the Council for Scientific & Industrial Research (CSIR)?

 ( a ) National Innovation Foundation
 ( b ) NITI Aayog
 ( c ) National Knowledge Commission
 ( d ) Ministry of AYUSH

67.Consider the following statements regarding government schemes in India

1)Centrally Sponsored Schemes are completely funded by the Union government and implemented by the Central Government machinery
2)Under Central Sector Schemes, a certain percentage of the funding is borne by the States and the implementation is by the State Governments.

Which of the statements above are true?

 (a) 1 only
 (b) 2 only
 (c) Both 1 and 2
 (d) Neither 1 nor 2

68.Which among the following is the objective of the UDAN scheme?

 (a ) Regional airport development and Regional Connectivity
 ( b ) Technical education of girls from tribal regions
 ( c ) Higher education of the single girl child
 ( d ) Modernisation of the civil aviation sector

69. Global Biodiversity Outlook is published by:

 ( a ) IUCN
 ( b ) TRAFFIC
 ( c ) UNEP
 ( d ) UNCBD

70. Public education and awareness is keyin achieving biodiversity targets. The efforts of the Hargilla Army, a conservation brigade of 70 local women in Assam, have been recognized internationally. Which among the following birds is the Hargilla Army conserving?

 ( a ) Greater Adjutant Stork
 ( b ) Jerdon’s Courser
 ( c ) Red-headed Vulture
 ( d ) White-bellied Heron

Q71. Consider the following statements about Intergovernmental Science-Policy Platform on Biodiversity and Ecosystem Services (IPBES):

1.It is an inter-governmental body that assesses the state of biodiversity and nature's contributions to people.
2. Its secretariat is located in Bonn.
3.It would publish the Global Assessment Report on Biodiversity in 2019.

Which among the above statements

is / are correct?

 ( a ) 1 and 2 only
 ( b ) 2 and 3 only
 ( c ) 1 and 3 only
 ( d ) 1 2 and 3

Q72.With respect to the Sur interregnum, consider the following statements:

1)Sher Shah waged extensive wars with the Rajputs and expanded his empire till Kashmir
2)Malik Muhammad Jayasi wrote the famous Hindi work Padmavat during his reign.

Which of the above statements is/are correct?

 ( a ) 1 only
 ( b ) 2 only
 ( c ) Both 1 and 2
 ( d ) Neither 1 nor 2

73. ISPATI IRADA sometimes seen in the news recently is the logo of which of the following?

a.   Ministry of Steel
b.   Food Corporation of India
c.   Indian Council for Agricultural Research
d.   National Bank for Agriculture and Rural Development

74. Consider the following statements with respect to Indian Sign Language Research and Training Centre (ISLRTC)

1. Its one of the main objective is to promote the use of Indian Sign Language as educational mode for deaf students at primary, secondary and higher education levels.

2. It is an autonomous body under the Department of Empowerment of Persons with Disabilities, Ministry of Social Justice and Empowerment.
Which of the statement(s) given above is/are correct?

a.   1 only
b.   2 only
c.   Both 1 and 2
d.   Neither 1 nor 2

75) Consider the following statements with respect to International Sign Language Day

1. The theme for this year is “Sign Language-Rights for all”.
2. The objective of celebrating the sign language day is to commemorate the birth anniversary of American author Helen Keller.

Which of the statement(s) given above is/are correct?

a.   1 only
b.   2 only
c.   Both 1 and 2
d.   Neither 1 nor 2

76) Which of the following is not one of the objective of UMMID’ initiative?

a.   To produce skilled clinicians in Human Genetics
b.   To monitor vaccination of children in the primary level
c.   To establish NIDAN Kendras to provide counselling in Government Hospitals wherein the influx of patients is more
d.   To undertake screening of pregnant women and new born babies for inherited genetic diseases in hospitals at aspirational districts

77) Which of the following are Kharif Crops?

1. Paddy
2. Cotton
3. Jowar
4. Wheat
5. Maize

Select the correct answer using the codes given below:

a.   All except 4
b.   All except 3 and 4
c.   1, 2 and 3 only
d.   All of the above

78) Consider the following statements with respect to Emissions Trading Scheme (ETS)

1. It is a market in which the traded commodity is allowed to trade in ‘carbon credits’.
2. It was launched by the Ministry of Environment, Forest and Climate Change.

Which of the statement(s) given above is/are correct?

a.   1 only
b.   2 only
c.   Both 1 and 2
d.   Neither 1 nor 2

79) Consider the following statements with respect to Participatory Guarantee Scheme (PGS)

1. It is a process of certifying organic products, which ensures that their production takes place in accordance with laid-down quality standards.
2. The certification is in the form of a documented logo or a statement.

Which of the statement(s) given above is/are correct?

a.   1 only
b.   2 only
c.   Both 1 and 2
d.   Neither 1 nor 2

80) Consider the following statements with respect to Microhyla eos sometimes seen in the news recently

1. It is a new species of frog found in Assam.
2. Microhyla are a group of narrow-mouthed frogs that is primarily and widely distributed in Western and Southern America.

Which of the statement(s) given above is/are correct?

a.   1 only
b.   2 only
c.   Both 1 and 2
d.   Neither 1 nor 2

81) Consider the following statements

1. It is the eastern-most protected area in the country.
2. It falls within the geographical sub-tropical zone and enjoys the sub-tropical climate.
3.  It is located at the junction of the Indian Sub-Continent Biogeograhic region and the Indo-China Biogeographic Region.

Identify the Tiger Reserve using the description given above:

a.   Pakke Tiger Reserve
b.   Sunderbans Tiger Reserve
c.   Namdapha Tiger Reserve
d.   Valmiki Tiger Reserve

82.) United in Science Report sometimes seen in the news recently was compiled by?

a.   World Science Forum
b.   World Economic Forum
c.   World Meteorological Organization
d.   UN Educational, Scientific and Cultural Organization

83. Consider the following statements with respect to National Service Scheme (NSS)

1. The main objective of the programme is to develop student’s personality through community service.
2. It comes under the aegis of Ministry of Home Affairs.

Which of the statement(s) given above is/are correct?

a.   1 only
b.   2 only
c.   Both 1 and 2
d.   Neither 1 nor 2

84.) Consider the following statements with respect to India International Science Festival (IISF) 2019

1. It is the fifth edition of IISF since its inception in 2015.
2. The theme for this year’s festival is RISEN India.
3. It is jointly organised by the Ministry of Science and Technology and Ministry of Earth Sciences.

Which of the statements given above are correct?

a.   1 and 2 only
b.   1 and 3 only
c.   2 and 3 only
d.   1, 2 and 3

85.) Consider the following statements with respect to Private Security Agency Licensing Portal

1. The portal will help in police verification of security guards.
2. The portal will be available only in English and Hindi.
3. It was launched by the Ministry of Home Affairs.

Which of the statement(s) given above is/are correct?

a.   3 only
b.   1 and 2 only
c.   1 and 3 only
d.   1, 2 and 3

86.)  Consider the following statements with respect to Global Goal Keeper Award

1. It is an annual award of Amnesty International.
2. The Award for the year 2019 was received by Indian Prime Minister Narendra Modi for his ambitious Beti Bachao Beti Padhao Scheme.

Which of the statement(s) given above is/are correct?

a.   1 only
b.   2 only
c.   Both 1 and 2
d.   Neither 1 nor 2

87.) Consider the following statements with respect to CHC Farm Machinery App

1. It will provide farmers the information of best demonstration of high-yielding crops and seeds in their nearby area.
2. It is a bi-lingual mobile app designed and developed by the Indian Council of Agricultural Research (ICAR).

Which of the statement(s) given above is/are correct?

a.   1 only
b.   2 only
c.   Both 1 and 2
d.   Neither 1 nor 2

88.) Consider the following statements with respect to Krishi Kisan App

1. Through the app, farmers can select and order the required machinery at the rates feasible for them from the Custom Hiring Centres located in the radius of 50 Kms.
2. The App will also help in geo-tagging and geo-fencing of crop and give weather forecast message to farmers.

Which of the statement(s) given above is/are correct?

a.   1 only
b.   2 only
c.   Both 1 and 2
d.   Neither 1 nor 2

89) Consider the following statements with respect to Senior Citizens Savings Scheme (SCSS)

1. It is a government-backed savings instrument offered to Indian residents aged over 60 years.
2. Under the Scheme, a senior citizen can deposit up to Rs 15 lakh rupees.
3. The interest on Senior Citizens Savings Scheme (SCSS) is fully taxable.

Which of the statement(s) given above is/are correct?

a.   1 only
b.   1 and 2 only
c.   2 and 3 only
d.   1, 2 and 3

90.) Consider the following statements with respect to Asperger’s Syndrome

1. It is a condition which results in continuous decline in thinking, behavioural and social skills that disrupts a person's ability to function independently.
2. The early signs of the disease may be forgetting recent events or conversations.

Which of the statement(s) given above is/are correct?

a.   1 only
b.   2 only
c.   Both 1 and 2
d.   Neither 1 nor 2

91.) Aqaba Process sometimes seen in the news recently is associated with which of the following?

a.   Fight against extremism
b.   Western Indian Ocean Region
c.   Medical Research Ethics
d.   Sustainable Development

92. Consider the following statements with respect to Financial Action Task Force (FATF)

1. It was established by a Group of Seven (G-7) Summit in Paris.
2. The FATF's decision making body, the FATF Plenary, meets three times per year. 

Which of the statement(s) given above is/are correct?

a.   1 only
b.   2 only
c.   Both 1 and 2
d.   Neither 1 nor 2

93.Which one of the following is the aim of Union government’s ‘mCessation Programme’, recently seen in news?

A.            Initiative to quit tobacco by utilising mobile technology
B.            Address the major causes of mortality among women and children
C.            Prevention and control of Vector Borne Diseases (VBDs)
D.            Early detection of Leprosy

94.With reference to recommendations made by RBI internal working group to review the current liquidity management framework, consider the following statements:

1. The current liquidity management framework should largely continue in its present form- a corridor system with the call money rate as the target rate.
2. There should be ideally one single overnight variable rate operation in a day.

Which of the statements given above is/are correct?

A.            1 only
B.            2 only
C.            Both 1 and 2
D.            Neither 1 nor 2

95.Which one of the following statements best describes the Section 35A of the Banking Regulation Act, 1949, recently seen in news?

A.    It vests power in the RBI to give directions to banks and can take action.
B.    It vests power in the central government to appoint the central bank governor.
C.    It empowers the central government to issue directions to the RBI in public interest.
D.    It enables RBI to prescribe CRR for scheduled banks without any floor or ceiling rate.

96. Joint Military Exercise KAZIND-2019, recently seen in news, is between India and which of the following countries?

A.            Kyrgyzstan
B.            Kenya
C.            Kiribati
D.            None of the above

97. With reference to the Rural Sanitation Strategy (2019-2029), consider the following statements:

1. It has been prepared by NITI Aayog.
2. It focuses on sustaining the sanitation behaviour change that has been achieved under the Swachh Bharat Mission Grameen (SBM-G).

Which of the statements given above is/are correct?

A.    1 only
B.    2 only
C.    Both 1 and 2
D.    Neither 1 nor 2

98. With reference to World Tourism Day, consider the following statements:

1. The theme for World Tourism Day 2019 is ‘Tourism and Jobs: a better future for all’.
2. The United Nations World Tourism Organisation (UNWTO) has selected India as the host country for official World Tourism Day 2019 celebrations.

Which of the statements given above is/are correct?

A.  1 only
B.  2 only
C.  Both 1 and 2
D.  Neither 1 nor 2

99. Which one of the following statements best describes ‘Fundamental Rule 56(J) of Central Civil Services (Pension) Rules, 1972’, recently seen in news?

A.    Periodical review of the performance of government servants with a view to ascertain whether they should be retained in service or retired in public interest.
B.    Pension once authorized after final assessment shall not be revised to the disadvantage of the Government servant.
C.    A Government servant can apply for voluntary retirement after completion of 30 years of qualifying service.
D.            None of the above.

100. With reference to the SASTRA Ramanujan Prize, consider the following statements:

1. The prize was instituted in 2005 by Union Ministry of Science and Technology.
2. The prize is conferred annually on mathematicians from across the world who are less than 32 years of age.

Which of the statements given above is/are correct?

A.  1 only
B.  2 only
C.  Both 1 and 2
D.  Neither 1 nor 2



ANSWER KEY


JATF LOGO-1.jpg

WEEKLY CURRENT AFF TEST ANSWER KEY 22 TO 28 SEPT 2019



Q.1) Solution (a) Cyberdome is a technological research and development centre of Kerala Police Department, conceived as a cyber centre of excellence in cyber security, as well as technology augmentation for effective policing. It envisages as a high tech public-private partnership centre of collaboration for different stakeholders in the domain of cyber security and handling of cyber crimes in a proactive manner. One of the main objectives of the Cyberdome is to prevent cyber crimes through developing a cyber threat resilient ecosystem in the state to defend against the growing threat of cyber attacks by synergizing with other departments and nodal agencies of the state. Cyberdome makes a collective coordination among the Government departments and agencies, academia, research groups, non-profitable organizations, individual experts from the community, ethical hackers, private organizations, and other law enforcement agencies in the country with an aim of providing a safe and secure cyber world for each and every citizen in the state. The primary objective of Cyberdome is to prevent cyber crimes and ensure that our cyber resources are secured. As of it is in place with the Assam and Kerala Police.

Q.2) Solution (c) The Caribbean Community (CARICOM or CC) is an organisation of fifteen Caribbean nations and dependencies having primary objectives to promote economic integration and cooperation among its members, to ensure that the benefits of integration are equitably shared, and to coordinate foreign policy. Currently CARICOM has 15 full members, 5 associate members and 8 observers. All of the associate members are British overseas territories.

Q.3 Solution (a) Abqaiq is a Saudi Aramco gated community and oil-processing facilities located in the Eastern Province of Saudi Arabia, located in the desert 60 km southwest of the DhahranDammam-Khobar metropolitan area, and north of the Rub' al-Khali, the second largest sand desert in the world also known as the "Empty Quarter".

Q.4) Solution (a) 1.5-Degree Report (SR 1.5) · It is an IPCC special report on the impacts of global warming of 1.5 °C above preindustrial levels and related global greenhouse gas emission pathways, in the context of strengthening the global response to the threat of climate change, sustainable development, and efforts to eradicate poverty. · Carbon dioxide emissions must reach net zero by 2050 in order to keep global warming below 1.5 degrees Celsius according to the report

Q.5) Solution (c) Shondol dance, which is known as the royal dance of Ladakh, has created history by entering into the Guinness Book of World Records as the largest Ladakhi dance. Shondol is a famous dance, which artistes used to perform for the king of Ladakh.

Q.6) Solution (a) Naropa Festival takes places every 12 years in the Tibetan calendar to celebrate the life of Buddhist philosopher and scholar Naropa.

Q.7) Solution (c) The Genetic Engineering Appraisal Committee (GEAC) is the apex body constituted in the Ministry of Environment and Forests under 'Rules for Manufacture, Use, Import, Export and Storage of Hazardous Microorganisms/Genetically Engineered Organisms or Cells 1989', under the Environment Protection Act, 1986. The Rules of 1989 also define five competent authorities i.e. the Institutional Biosafety Committees (IBSC), Review Committee of Genetic Manipulation (RCGM), Genetic Engineering Approval Committee (GEAC), State Biotechnology Coordination Committee (SBCC) and District Level Committee (DLC) for handling of various aspects of the rules.

Q.8) Solution (c) PGS is a process of certifying organic products, which ensures that their production takes place in accordance with laid-down quality standards. The certification is in the form of a documented logo or a statement. PGS is a “quality assurance initiative that is locally relevant, emphasize[s] the participation of stakeholders, including producers and consumers, and (which) operate[s] outside the framework of third-party certification”. Individual farmers or group of farmers smaller than five members are not covered under PGS. They either have to opt for third party certification or join the existing PGS local group.

Q.9) Solution (c) In 2013, the Governments of the Republic of India and the United States of America launched Promoting Energy Access through Clean Energy (PEACE) as a new track under the U.S.-India Partnership to Advance Clean Energy (PACE), alongside the existing research (PACE-R) and deployment (PACE-D) tracks. PEACE aims to harness commercial enterprise to bring clean energy to individuals and communities un-served and underserved by the electricity grid. In June 2015, these governments established the PACEsetter Fund, a new joint fund to support the PEACE initiative by providing early-stage grant funding to accelerate the commercialization of innovative off-grid clean energy products, systems, and business models.
The mission of the PACEsetter Fund is to accelerate the commercialization of innovative offgrid clean energy access solutions by providing early-stage grant funding that would allow businesses to develop and test innovative products, business models and systems. The Fund's main purpose is to improve the viability of off-grid renewable energy businesses that sell small scale (under 1 megawatt) clean energy systems to individuals and communities without access to grid connected power or with limited/intermittent access . (less than 8 hours per day).

Q.10) Solution (b)
The Global Internet Forum to Counter Terrorism (GIFCT), established by Facebook, Microsoft, Twitter and YouTube with the objective of disrupting terrorist abuse on their respective platforms, will now become an independent organization led by an Executive Director and supported by dedicated technology, counterterrorism and operations teams.

Q.11) Solution (a) A new initiative was launched on 23rd September at the UN Climate Action Summit to help guide the world’s heaviest greenhouse gas emitting industries toward the low-carbon economy. India and Sweden together with Argentina, Finland, France, Germany, Ireland, Luxembourg, the Netherlands, South Korea and the UK, as well as a group of companies including Dalmia Cement, DSM, Heathrow Airport, LKAB, Mahindra Group, Royal Schiphol Group, Scania, SpiceJet, SSAB, ThyssenKrupp and Vattenfall, announced a new Leadership Group for Industry Transition that will drive transformation in hard-to-decarbonize and energyintensive sectors. This global initiative will be supported by the World Economic Forum, the Energy Transitions Commission, Mission Innovation, Stockholm Environment Institute, and the European Climate Foundation among many others in an ambitious, public-private effort, to ensure heavy industries and mobility companies can find a workable pathway to deliver on the Paris Agreement.

Q.12) Solution (d) Centralized Public Grievance Redress And Monitoring System (CPGRAMS) is an online webenabled system over NICNET developed by NIC, in association with Directorate of Public Grievances (DPG) and Department of Administrative Reforms and Public Grievances (DARPG). CPGRAMS is the platform based on web technology which primarily aims to enable submission of grievances by the aggrieved citizens from anywhere and anytime (24x7) basis to Ministries/Departments/Organisations who scrutinize and take action for speedy and favourable redress of these grievances. Tracking grievances is also facilitated on this portal through the system generated unique registration number.

Q.13) Solution (c) Prime Minister Narendra Modi received the 2019 Global Goalkeeper Award for Swachh Bharat Mission from Bill and Melinda Gates Foundation. Global Goalkeeper Award is a "special recognition" by the Bill and Melinda Gates Foundation. The award celebrates a political leader who has "demonstrated their commitment to the Global Goals through impactful work in their country and/or globally."

Q.14) Solution (a) ISRO, is developing methane-powered rocket engines. Methane, which can be synthesised with water and carbon dioxide in space, is often described as the space fuel of the future. ISRO is developing two ‘LOx methane’ engines (liquid oxygen oxidiser and methane fuel) engines. One of the two projects is trying to convert the existing cryogenic engine, which uses liquid hydrogen for fuel, into a LOx methane engine. The other is a smaller engine of 3 tonnes thrust, which will feature an electric motor. ISRO currently prefers to use a fuel called Unsymmetrical Di-Methyl Hydrazine, along with Nitrogen tetroxide for oxidiser, in its liquid fuel (Vikas) engines, which are used in the lower stages of its rockets, PSLV and GSLV. This fuel, like all hydrazine-based fuels, is said to be highly toxic and cancer-causing. Globally, governments are keen on banning hydrazine. Besides, methane beats hydrazine on every other count, too. Apart from being non-toxic, it has a higher specific impulse (which means one kg of the gas can life one kg of mass for a longer time), it is easy to store, does not leave a residue upon burning, less bulky, and, importantly, can be synthesised up in space

15. Solution: c)

The Financial Action Task Force (FATF) is an inter-governmental body established in 1989 on the
initiative of the G7. The objectives of the FATF are to set standards and promote effective implementation of legal, regulatory and operational measures for combating money laundering, terrorist financing and other related threats to the integrity of the international financial system.

The FATF is therefore a “policy-making body” which works to generate the necessary political will to bring about national legislative and regulatory reforms in these areas.

The FATF has developed a series of Recommendations that are recognised as the international standard for combating of money laundering and the financing of terrorism and proliferation of weapons of mass destruction.

The FATF monitors the progress of its members in implementing necessary measures, reviews money laundering and terrorist financing techniques and counter-measures, and promotes the adoption and implementation of appropriate measures globally.


16. Solution: d)

India’s candidature for a non-permanent seat in the Security Council for the 2021-22 term has been endorsed unanimously by the Asia Pacific group, which comprises 55 countries, including Pakistan.
India has been a non-permanent member of the Security Council seven time previously: 1950-51, 1967-68, 1972-73, 1977-78, 1984-85, 1991-92 and 2011-12.
Each year, the General Assembly elects five non-permanent members out of a total of 10, for a two-year term.
Distribution of seats: These 10 seats are distributed among the regions thus: five for African and Asian countries; one for Eastern European countries; two for Latin American and Caribbean countries; two for Western European and other countries.


17. Solution: a)

What are Total Polar Compounds (TPC)?
  1. In many countries, TPC is used to measure the quality of oil. The level of TPC increases every time oil is re-heated. Some of the studies show that TPC accumulation in oil without food is slower than that in oil frying with food.
  2. Higher level of TPC in cooking oil leads to health issues like hypertension, atherosclerosis, Alzheimer’s disease and liver disease. One of the studies also noticed high levels of glucose, creatinine and cholesterol with declined levels of protein and albumin in cooking oil.

18. Solution: c)
 Helminths (worms) which are transmitted through soil contaminated with faecal matter are called soil-transmitted helminths (Intestinal parasitic worms).

STH infections can lead to anemia, malnutrition, impaired mental and physical & cognitive development, and reduced school participation.

STH Infections can be prevented by:
  • Using sanitary toilets, not defecating outside
  • Hand-washing, particularly before eating and after using toilets
  • Wearing slippers and shoes
  • Washing fruits and vegetables in safe and clean water
  • Eating properly cooked food
 Objective of National Deworming Day:
The objective of National Deworming Day is to deworm all preschool and school-age children (enrolled and non-enrolled) between the ages of 1-19 years through the platform of schools and Anganwadi Centers in order to improve their overall health, nutritional status, access to education and quality of life.
The Ministry of Health & Family Welfare, Government of India is the nodal agency for providing all States/UTs with guidelines related to National Deworming Day (NDD) implementation at all levels.

19. Solution: c)

The Central Vigilance Commission (CVC) has constituted an ‘Advisory Board for Banking Frauds (ABBF)’ to examine bank fraud of over 50 crore and recommend action.
  1. The board’s jurisdiction would be confined to those cases involving the level of officers of General Manager and above in the Public Sector Banks in respect of an allegation of fraud in a borrowal account.
  2. It would function as the first level of examination of all large fraud cases before recommendations or references are made to the investigative agencies by the respective public sector banks (PSBs).
  3. Lenders would refer all large fraud cases above 50 crore to the board and on receipt of its recommendation or advice, the bank concerned would take further action in such matter.
  4. The Central Bureau of Investigation may also refer any case or matter to the board where it has any issue or difficulty or in technical matters with the PSB concerned.
  5. It would also periodically carry out frauds analysis in the financial system and give inputs for policy formulation related to the fraud to the RBI.

20. Solution: a)

As part of centenary year celebrations of Dr. Vikram Sarabhai, father of Indian space programme, ISRO has announced its “Vikram Sarabhai Journalism Award in Space Science, Technology and Research”.
The award recognizes and rewards journalists who have actively contributed towards the field of space science, applications, and research.
  1. Vikram Sarabhai set up India’s first rocket launch site in Thumba, a small village near the Thiruvananthapuram airport in Kerala.
  2. Vikram Sarabhai was also responsible for bringing cable television to India. His constant contact with NASA paved a way for the establishment of Satellite Instructional Television Experiment (SITE) in 1975.
  3. Sarabhai was the mastermind behind building India’s first satellite, Aryabhata.
  4. He was one of the founding members of the Indian Institute of Management, Ahmedabad (IIMA).

21. Solution: b)

There was 41 per cent reduction in crop residue burning in North India in 2018 compared with that in 2016. As many as 4,500 villages in Haryana and Punjab were declared zero stubble burning villages in 2018.
Under a central government scheme for promoting agricultural mechanisation for in-situ management of crop residue in North India between 2018-19 and 2019-20, an amount of 1,151.80 crore has been allocated. Within the first year of implementation, the Happy Seeder/zero tillage technology was adopted in 8 lakh hectares of land in these States.
Stubble burning results in emission of harmful gases such carbon dioxide, sulphur dioxide, nitrogen dioxide along with particulate matter.


22. Solution: b)

Global Environment Outlook report has been released. The report is the sixth and is the UN’s most comprehensive report on the state of the global environment since the fifth edition in 2012.
  • The GEO is often referred to as UN Environment’s flagship environmental assessment.
  • The first publication was in 1997 and was originally requested by Member States.
GEO global assessments provide an integrated analysis (e.g. social, economic, environmental) of major trends that have shaped the environment. These reports provide world leaders with policy options to take immediate action to address environmental issues by turning environmental discussions into practice.


23. Solution: b)

The 2019 G20 Osaka summit was the fourteenth meeting of the G20, a forum of 19 countries and the EU that together represent most of the world economy. It was the first G20 summit to be hosted by Japan.

The 19 countries are Argentina, Australia, Brazil, Canada, China, France, Germany, India, Indonesia, Italy, Japan, Mexico, Republic of Korea, Republic of South Africa, Russia, Saudi Arabia, Turkey, United Kingdom, United States of America.

The recent G20 summits have focused not only on macroeconomy and trade, but also on a wide range of global issues which have an immense impact on the global economy, such as development, climate change and energy, health, counter-terrorism, as well as migration and refugees.

24. Solution: c)

The Ministry of Environment, Forest and Climate Change (MoEFCC) recently designated the Council of Scientific and Industrial Research-National Physical Laboratory (CSIR-NPL) as a national agency that shall be responsible for carrying out certification for instruments and equipments for monitoring emissions and ambient air.
The ministry issued a notification under Section 3 of the Environment Protection Act, giving CSIR-NPL the authority to certify instruments.
The government launched National Clean Air Programme, a time-bound national level strategy to tackle increasing air pollution.

25. Solution: c)

Under a negative rate policy, financial institutions are required to pay interest for parking excess reserves with the central bank.
That way, central banks penalise financial institutions for holding on to cash in hope of prompting them to boost lending.

What are the pros of negative rates?
  1. Lowers borrowing costs.
  2. Help weaken a country’s currency rate by making it a less attractive investment than that of other currencies.
  3. A weaker currency gives a country’s export a competitive advantage and boosts inflation by pushing up import costs.
26. Solution: c)

World Wide Fund for Nature (WWF) has released the first-ever global assessment of forest biodiversity.

Key findings:
  • There has been a 53% decline in the number of forest wildlife populations since 1970.
  • Of the 455 monitored populations of forest specialists, more than half declined at an annual rate of 1.7 per cent, on average between 1970 and 2014.
  • While the decline was consistent in these years among mammals, reptiles and amphibians (particularly from the tropical forests), it was less among birds (especially from temperate forests).

27. Solution: c)

India’s first and indigenous microprocessor “Shakti” developed by Indian Institute of Technology Madras (IITM).

28. Solution: c)

C-Sat-Fi (C-DOT Satellite WiFi) is based on the optimal utilization of wireless and satellite communication to extend connectivity to the unserved areas including the remote islands and difficult terrains. Besides offering the ease of deployment, the solution is ideally suited to addressing disasters and emergencies when no other means of communication are available. This cost-effective solution does not require the expensive Satellite Phones and can work on any WiFi enabled phone.


29. Solution: d)

According to the IMF’s “Annual Observance Report of the Special Data Dissemination Standard for 2018”, India failed to comply with multiple requirements prescribed in the Special Data Dissemination Standard (SDDS).

Special Data Dissemination Standard (SDDS) is an International Monetary Fund standard to guide member countries in the dissemination of national statistics to the public.
It was established in April 1996.

30. Solution: d)
 A notifiable disease is any disease that is required by law to be reported to government authorities.
The process helps the government keep track and formulate a plan for elimination and control.
The onus of notifying any disease and the implementation lies with the state government.
Any failure to report a notifiable disease is a criminal offence and the state government can take necessary actions against defaulters.
The Centre has notified several diseases such as cholera, diphtheria, encephalitis, leprosy, meningitis, pertussis (whooping cough), plague, tuberculosis, AIDS, hepatitis, measles, yellow fever, malaria dengue, etc.

31. Solution: b)
 Know India Programme is a flagship programme of Ministry of External Affairs for engagement with Indian origin youth (between 18-30 years) to enhance their awareness about India, its cultural heritage, art and to familiarise them with various aspects of contemporary India.

Eligibility: Minimum qualification required for participating in KIP is graduation from a recognized University /Institute or enrolled for graduation and ability to speak in English. The applicant should not have visited India through any previous Programme of Government of India. Those who have not visited India before will be given preference.

32. Solution: d)

The objectives of the Central Consumer Protection Council is to promote and to protect the rights of the consumers such as:-
  1. The right to be protected against the marketing of goods and services which are hazardous to life and property.
  2. The right to be informed about the quality, quantity, potency, purity, standard and price of goods or services, as the case may be so as to protect the consumer against unfair trade practices;
  3. The right to be assured, wherever possible, access to a variety of goods and services at competitive prices;
  4. The right to be heard and to be assured that consumer’s interest will receive due consideration at appropriate forums;
  5. The right to seek redressal against unfair trade practices or restrictive trade practices or unscrupulous exploitation of consumers; and
  6. The right to consumer education.


33. Solution: d)
 Chandrayaan-2’s orbiter or mother spacecraft has zeroed in on a crater on the moon named after 20th century’s acclaimed radio physicist Sisir Kumar Mitra. The Mitra crater is on the edge of another crater.


34. Answer: b
Explanation:
  • Other than offenses of terrorism, it also deals with counterfeit currency, human trafficking, narcotics or drugs, organized crime (extortion mobs and gangs), plane hijacking and violations of atomic energy act and weapons of mass destruction act.
  • It extends to  citizens of India outside India

35. Answer: c
Explanation:
  • It Started as a movement among a few people in lower middle-class rural France protesting a new eco-tax on fuel they felt would push their budgets over the edge.
  • The movement was Galvanized by rising fuel prices, the high cost of living and claims that a disproportionate burden of the government’s tax reforms were falling on the working and middle classes (especially those in rural and peri-urban areas)
  • So protesters have called for reductions in fuel taxes, the reintroduction of the solidarity tax on wealth, the raising of the minimum wage, and the resignation of the President of France, Emmanuel Macron.
  • It is named after the yellow high-visibility jackets French motorists must carry in their vehicles.

36. Answer: d
Explanation:
  • IMMSAREX:- IONS Multilateral Maritime Search and Rescue Exercise (IMMSAREX),
  • It comes under the aegis of Indian Ocean Naval Symposium (IONS)

37. Answer: a
Explanation:
“Khaan Quest” originally began as a joint exercise between the Mongolian Armed Forces and the U.S. Pacific Command. It became an international military exercise in 2006. The international exercise “Khan Quest” was initially started in 2003 by the United States and Mongolia. At present, India also participates in “Khan Quest”, besides “Nomadic Elephant” which is a bilateral exercise between the two countries. It brings together over a dozen foreign militaries to engage in the sharing of practices for multinational peacekeeping operations. In the exercise, personnel gain United Nations peacekeeper training as well as certification for support of peacekeeping operations. India and Mongolia at carry out regular bilateral military exercise named “Nomadic Elephant” since the year 2004.

38. Answer: d
Explanation:
Fiscal Deficit = Budgetary Deficit + Borrowings and Other Liabilities of the government. The difference between total revenue and total expenditure of the government is termed as fiscal deficit. It is an indication of the total borrowings needed by the government. The fiscal deficit is the primary deficit plus interest payments on the debt. Fiscal responsibility and Budget Management Act concerns both Fiscal and Revenue deficit.


39. Answer: c
Explanation:
The Goods and Services Tax (GST) is governed by the GST Council – a constitutional body. Article 279 (1) of the amended Indian Constitution states that the GST Council has to be constituted by the President within 60 days of the commencement of the Article 279A. In the GST Council, the Centre would have 1/3rd of the voting strength and the remaining 2/3rd voting strength vests with the states. Central Government will notify the rate of tax to be levied under the Central Goods and Services Tax Act as per the recommendations of the GST Council.


40. Answer: a
Explanation:
Biomining is the process of using microorganisms to extract metals of economic interest from rock ores or mine waste. Biomining techniques may also be used to clean up sites that have been polluted with metals. When the metal of interest is directly dissolved, the biomining process is called “bioleaching,” and when the metal of interest is made more accessible or enriched in the material left behind, it is called “biooxidation”. Both processes involve microbial reactions that can happen anywhere the microbes, rocks, and necessary nutrients, like oxygen, occur together.
Bioremediation is the use of either naturally occurring or deliberately introduced microorganisms to consume and break down environmental pollutants, in order to clean a polluted site.

41. Answer: d
Explanation:
The WHO Framework Convention on Tobacco Control (WHO FCTC) is the first international public health treaty negotiated under the auspices of the WHO. The objective of FCTC is to provide a framework for supply and demand reduction measures for tobacco control at the national, regional and global levels. The WHO FCTC is an evidence based, legally binding multilateral treaty with 181 parties, and one of the most widely ratified treaties in the UN system.  Govt. of India ratified the WHO Framework Convention on Tobacco Control (WHO FCTC) in 2004.

42. Answer: b
Explanation:
Naga Chilli (Capsicum chinense) also known as King Chilli, is more popular as Bhut/Bhoot Jolokia. It derives its name from the popular belief that it originated in the hills of Bhutan. The Guinness Book of World Records had in 2007 named it as the hottest chilli but lost its position first to a U.K. variety called Infinity and then the U.S.­developed Carolina Reaper. The chilli has now been used by a Naga panchakarma specialist in India for an acupuncture theory.

43. Answer: d
Explanation:
The Unique Identification Authority of India or UIDAI is an agency under the central government of India mandated to collect demographic and biometric information of the country’s residents, store the data in a central database, and issue to each resident of the country a 12-digit unique identity number called Aadhaar. The Aadhaar Act, 2016 states that every resident shall be entitled to obtain an aadhaar number. The Act further defines residency as an individual who has resided in India for a period or periods amounting in all to 182 days or more in the 12 months immediately preceding the date of application for enrolment. UIDAI functions under the Ministry of Electronics and Information Technology.

44.Answer: c
Explanation:
Sundarban region of West Bengal and other ecologically sensitive areas identified are as under Environment (Protection) Act, 1986 such as Gulf of Khambat and Gulf of Kutchh in Gujarat, Malvan, Achra-Ratnagiri in Maharashtra, Karwar and Coondapur in Karnataka, Vembanad in Kerala, Gulf of Mannar in Tamil Nadu, Bhitarkanika in Odisha, Coringa, East Godavari and Krishna in AP are treated as Critical Vulnerable Coastal Areas (CVCA) and managed with the involvement of coastal communities including fisherfolk who depend on coastal resources for their sustainable livelihood.


45. Answer: a
Explanation:
The consolidated goal on health is SDG 3. One of these targets, (Target 3.3), explicitly mentions TB. SDG 3 also includes a target (Target 3.8) related to universal health coverage (UHC) in which TB is explicitly mentioned. This includes an indicator on the coverage of essential prevention, treatment and care interventions. WHO’s End TB strategy serves as a blueprint to countries to reduce TB incidence by 80%, TB deaths by 90% and to eliminate catastrophic costs for TB affected households by 2030. The Government of India is committed to achieving the target of TB elimination by 2025, 5 years ahead of the WHO targets.


46. Answer: (b)
Difficulty Level: Easy
Explanation:
SITMEX-19 is a five day maritime exercise that aims to strengthen relations between Singapore-India-Thailand. It was organised between Indian Navy, Royal Thai Navy (RTN) and Republic of Singapore Navy (RSN). Indian Navy’s Ships Ranvir, a Missile Corvette, Sumedha, a Guided Missile Destroyer, Kora, and an offshore Patrol Vessel participated in the exercise. 

47. Answer: (d)
Difficulty Level: Medium
Explanation:
10 agencies which will be able to access NATGRID data on real time basis are:
1. Intelligence Bureau (IB)
2. Research & Analysis Wing (R&AW)
3. Central Bureau of Investigation (CBI)
4. Enforcement Directorate (ED)
5. Directorate of Revenue Intelligence (DRI)
6. Financial Intelligence Unit (FIU)
7. Central Board of Direct Taxes (CBDT)
8. Central Board of Excise and Customs (CBEC)
9. Directorate General of Central Excise and Intelligence (DGCEI)
10. Narcotics Control Bureau (NCB)
Initially, no state agencies will be given direct access to NATGRID data but in case any relevant information is required, they can approach NATGRID through any of 10 user agencies.

48. Answer: c
Difficulty Level: Medium
Explanation:
·Ayushman Bharat Yojana or Pradhan Mantri Jan Arogya Yojana (PMJAY) is a centrally sponsored scheme launched in 2018, under the Ayushman Bharat Mission of MoHFW in India. It is an umbrella of two major health initiatives namely, Health and Wellness centres and National Health Protection Scheme (NHPS)
·The National Health Protection Scheme (NHPS) scheme is formed by integrating multiple schemes including Rashtriya Swasthya Bima Yojana, Senior citizen health Insurance Scheme (SCHIS), Central Government Health Scheme (CGHS), Employees' State Insurance Scheme (ESIS) etc.
·Ayushman Bharat-National Health Protection Scheme, which will cover about 50 crore beneficiaries providing coverage up to 5 lakh rupees per family per year for secondary and tertiary care hospitalization.
·Benefits of the scheme are portable across the country and a beneficiary covered under the scheme will be allowed to take cashless benefits from any public or private empanelled hospitals across the country.
·It will be an entitlement-based scheme with entitlement decided on the basis of deprivation criteria in the Socio-Economic Caste Census (SECC) database.

49. Answer: c
Difficulty Level: Medium
Explanation:
·Swachh Sarvekshan, commissioned by Ministry of Urban Development and carried out by Quality Council of India, is an extensive sanitation survey across several hundred cities to check the progress and impact of Swachh Bharat Abhiyan and to foster a spirit of competition among the cities. The performance of each city is evaluated on six parameters:
o   Municipal solid waste, sweeping, collection and transportation
o   Municipal solid waste, processing and disposal of solid waste
o   Open defecation free/toilets
o   Capacity building and eLearning
o   Provision of public & community toilet seats
o   Information, education and communication, and behaviour change
· The Quality Council of India (QCI) is a pioneering experiment of the Government of India in setting up organizations in partnership with the Indian industry. QCI plays a pivotal role in propagating, adoption and adherence to quality standards in all important spheres of activities including education, healthcare, environment protection, governance, social sectors, infrastructure sector and such other areas of organized activities that have significant bearing in improving the quality of life and wellbeing of the citizens of India.
·The Ministry of Urban Development ranks cities based on cleanliness index. This list summarises the cities topping those lists annually.
·According to the Swachh Survekshan 2018 (cleanliness index is based on this), Indore in Madhya Pradesh has retained its title of being India's cleanest city. Bhopal has been ranked as India's second cleanest city for two years in a row and Chandigarh was ranked as the third cleanest city, moving up from Rank 11 in 2017.

50. Answer: b
Difficulty Level: Hard
Explanation:
· The model was developed by National Bank for Agriculture and Rural Development. Various commercial banks, state co-operative banks and regional rural banks are the participatory members who provide the card. A Kisan credit card will help the farmer to secure timely credit at reasonable rates of interest in a hassle-free manner. The farmer won’t be required to appear for the continuous screening process which is undertaken by the banks for giving term loans. A farmer can take maximum advantage of a Kisan credit card. He will be provided with a passbook which will have his name, credit limit, photograph, validity and details of his land holding.

51. Answer: a
Difficulty Level: Medium
Explanation:
·Interest subvention is a subsidy offered on interest rates. For example, the interest subvention scheme for farmers aims at providing short term credit to farmers at subsidized interest rate.
·Thus, interest subvention is a form of waiver of some percentage of interest that promotes some particular industry and general public interest. The government sets apart an amount in every budget to finance interest subversion.
·Government of India has come up with an array of interest subvention schemes for different sectors spanning from agriculture, education, handlooms, export-oriented sectors to housing sectors. Interest subvention schemes are implemented with the help of funds allocated by the government for the purpose in a particular financial year.

52. Answer: a
Difficulty level: Medium
Explanation
•Mahendravaraman I composed the short one-act Sanskrit play Mattavilasaprahasanam
• Mattavilasa Prahasana is a satire that pokes fun at the peculiar aspects of the heretic Kapalika and Pasupata Saivite sects, Buddhists and Jainism. The setting of the play is Kanchipuram, the capital city of the Pallava kingdom in the seventh century.

53. Answer: c
Difficulty level: Medium
Explanation
•The second stage of Pallava architecture is represented by the monolithic rathas and Mandapas found at Mamallapuram.
•Narasimhavarman I took the credit for these wonderful architectural monuments. The five rathas, popularly called as the Panchapanadava rathas, signifies five different styles of temple architecture.
•The mandapas contain beautiful sculptures on its walls. The most popular of these mandapas are Mahishasuramardhini Mandapa, Tirumurthi Mandapam and Varaha Madapam.
•The Pallavas had also contributed to the development of sculpture. Apart from the sculptures found in the temples, the ‘Open Art Gallery’ at Mamallapuram remains an important monument bearing the sculptural beauty of this period.
•The Descent of the Ganges and the Penance of Arjuna are fresco paintings in stone.

54. Answer: a
Difficulty level: Hard
Explanation
•The Chalukya administration was highly centralized unlike that of the Pallavas and the Cholas. Village autonomy was absent under the Chalukyas.
•The Chalukyas had great maritime power. Pulakesin II had 100 ships in his navy. They also had a small standing army.
•The Badami Chalukyas were Brahmanical Hindus but they gave respect to other religions. Importance was given to Vedic rites and rituals.
•The founder of the dynasty Pulakesin I performed the ashvamedha sacrifice. A number of temples in honour of Vishnu, Siva and other gods were also built during this period.


55. Answer: d
Difficulty Level: Medium
Explanation:
Department of Biotechnology of Ministry of Science and Technology has started the UMMID Initiative which is designed on the concept of ‘Prevention is better than Cure’. With a very large population and high birth rate, and consanguineous marriage favored in many communities, prevalence of genetic disorders is high in India, the UMMID initiative aims (i) to establish NIDAN Kendras to provide counselling, prenatal testing and diagnosis, management, and multidisciplinary care in Government Hospitals wherein the influx of patients is more, (ii) to produce skilled clinicians in Human Genetics, and (iii) to undertake screening of pregnant women and new born babies for inherited genetic diseases in hospitals at aspirational districts.
Source:

56. Answer: (c)
Difficulty Level: Medium
Explanation:
India and the United States are set to hold their first tri-services exercise code-named "Tiger Triumph" at Visakhapatnam and Kakinada in November this year. For the first time, the US and India will be holding the tri-service military exercise. India earlier conducted such tri-service exercise with Russia.

57. Answer: c
Difficulty level: Medium
Explanation
•Prime Minister Narendra Modi released a commemorative coin and stamp on Guru Gobind Singh to mark the 350th birth anniversary.
•Guru Gobind Singh was the tenth and last of the human Gurus of Sikhism. He became Guru at the age of 9 in 1675 after the martyrdom of his father, the ninth Guru, Guru Tegh Bahadur.
•Guru Gobind Singh played a vital role in moulding the Sikh religion into its present shape, with the institution of the Khalsa fraternity, and the completion of the sacred scripture, the Guru Granth Sahib. Guru Gobind Singh lost his four sons died during his lifetime two in battle and two executed by the Mughal army. Later he himself declared Guru Granth Sahib as the next and perpetual Guru of the Sikhs before leaving his mortal body in 1708.

58. Answer: c
Difficulty level: Easy
Explanation
•The founder of the Bahmani kingdom was Alauddin Bahman Shah also known as Hasan Gangu in 1347. Its capital was Gulbarga.
•There were a total of fourteen Sultans ruling over this kingdom. Among them, Alauddin Bahman Shah, Muhammad Shah I and Firoz Shah were important. Ahmad Wali Shah shifted the capital from Gulbarga to Bidar.
•By the year 1526, the Bahmani kingdom had disintegrated into five independent sultanates. They were Ahmadnagar, Bijapur, Berar, Golkonda and Bidar and known as Deccan Sultanates.

59. Answer – A
Difficulty Level: Medium
Explanation:
The International Union for Conservation of Nature (IUCN) is the world's main authority on the conservation status of species. The IUCN Red List of Threatened Species is the world's most comprehensive inventory of the global conservation status of biological species. Categories include:
• Extinct (EX) - No known individuals remaining
• Extinct in the wild (EW) - Known only to survive in captivity, or as a naturalized population outside its historic range
• Critically endangered (CR) - Extremely high risk of extinction in the wild
• Endangered (EN) - High risk of extinction in the wild
• Vulnerable (VU) - High risk of endangerment in the wild
• Near threatened (NT) - Likely to become endangered in the near future
• Least concern (LC) - Lowest risk


60. Answer – B
Difficulty Level: Medium
Explanation:
The Convention on Wetlands, signed in Ramsar, Iran, in 1971 – known as Ramsar Convention - is an intergovernmental treaty which provides the framework for national action and international cooperation for the conservation and wise use of wetlands and their resources. The Ramsar Convention was developed as a means to call international attention to the rate at which wetland habitats were disappearing, in part due to a lack of understanding of their important functions, values, goods and services.
Montreux Record is a register of wetland sites on the list of wetlands of international importance where changes in ecological character have occurred or are occurring, as a result of technological developments, pollution or other human interference. It is maintained as part of Ramsar List.
Currently, two wetlands of India are in Montreux record viz. Keoladeo National Park, Rajasthan and Loktak Lake, Manipur. Further, ChilikaLake was placed in the record but was later removed from it as it was reclaimed due to conservation efforts.

61. Answer – C
Difficulty Level: Medium
Explanation
Sea turtles occupy a unique position within the food web. They consume many preys such as sea grasses and algae. They also transport nutrients from marine areas such as sea-grass beds to energy-poor habitats like sandy beaches - this helps reverse the usual flow of nutrients from land to sea. They also help maintain the health of coral reefs and sea grass beds.

62. Answer: c
Difficulty Level: Medium
Explanation:
·The National Pension System (NPS) is a pension system administered and regulated by the Pension Fund Regulatory and Development Authority (PFRDA), created by an Act of the Parliament of India.
·On 10 December 2018, Government of India made withdrawals from the NPS an entirely tax-free instrument.
·In the 2019-20 budget, the mandatory contribution by the Central Government for employees covered under NPS has been enhanced from the existing 10% of salary to 14%. The hike in the government’s contribution will make NPS better than the defined pension under the old system where the pensioner got 50% of his last drawn salary. 

63. Answer: d
Difficulty Level: Easy
Explanation:
·Under the PM Ujjwala Yojana, the government aims at providing LPG connections to BPL households in the country. The scheme is aimed at replacing the unclean cooking fuels mostly used in the rural India with the clean and more efficient LPG (Liquefied Petroleum Gas).
·PMUY is likely to result in an additional employment of around 1 Lakh and provide business opportunity of around Rs. 10,000 Cr. over the next 3 Years to the Indian Industry. Launch of this scheme is also expected to provide a great boost to the ‘Make in India’ campaign as all the manufacturers of cylinders, gas stoves, regulators, and gas hose are domestic.

64. Answer: d
Difficulty Level: Medium
Explanation:
Department of Biotechnology of Ministry of Science and Technology has started the UMMID Initiative which is designed on the concept of ‘Prevention is better than Cure’. With a very large population and high birth rate, and consanguineous marriage favored in many communities, prevalence of genetic disorders is high in India, the UMMID initiative aims (i) to establish NIDAN Kendras to provide counselling, prenatal testing and diagnosis, management, and multidisciplinary care in Government Hospitals wherein the influx of patients is more, (ii) to produce skilled clinicians in Human Genetics, and (iii) to undertake screening of pregnant women and new born babies for inherited genetic diseases in hospitals at aspirational districts.

65. Answer: (c)
Difficulty Level: Medium
Explanation:
India and the United States are set to hold their first tri-services exercise code-named "Tiger Triumph" at Visakhapatnam and Kakinada in November this year. For the first time, the US and India will be holding the tri-service military exercise. India earlier conducted such tri-service exercise with Russia.

66. Answer: d
Difficulty Level: Medium
Explanation:
•The Ministry of AYUSH has established Traditional Knowledge Digital Library (TKDL) in collaboration with Council for Scientific & Industrial Research (CSIR) as an initiative to prevent misappropriation of the country’s traditional medicinal knowledge at International Patent Offices on which healthcare needs of more than 70% population and livelihood of millions of people in India is dependent.
•TKDL integrates diverse disciplines and languages such as Ayurveda, Unani, Siddha, Yoga, Sanskrit, Arabic, Urdu, Persian, Tamil, English, Japanese, Spanish, French, German, modern science & modern medicine.
•At present, as per the approval of Cabinet Committee on Economic Affairs, access of TKDL is available to nine International Patent Offices (European Patent Office, United State Patent & Trademark Office, Japan Patent Office, United Kingdom Patent Office, Canadian Intellectual Property Office, German Patent Office, Intellectual Property Australia, Indian Patent Office and Chile Patent Office), under TKDL Access (Non-disclosure) Agreement.

67. Answer: d
Difficulty Level: Medium
Explanation:
•In India there are two types of schemes viz; central sector and centrally sponsored scheme. The nomenclature is derived from the pattern of funding and the modality for implementation.
•Central sector schemes, are 100% funded by the Union government and implemented by the Central Government machinery. Central sector schemes are mainly formulated on subjects from the Union List.In addition, the Central Ministries also implement some schemes directly in States/UTs which are called Central Sector Schemes but resources under these Schemes are not generally transferred to States.

68. Answer: a
Difficulty Level: Easy
Explanation:
• UDAN (Ude Desh ka Aam Naagrik) scheme is aimed at regional airport development and "Regional Connectivity Scheme" (RCS). The main objectives are to enable the common citizen of the country to fly, making air travel affordable and widespread, to boost inclusive national economic development, job growth and air transport infrastructure development of all regions and states of India.
• The scheme has two components. The first component is to develop new airports and enhance the existing regional airports to increase the number of operational airports for scheduled civilian flights.

69. Answer D
Difficulty Level: Easy
Explanation
Global Biodiversity Outlook (GBO) is the flagship publication of the UN Convention on Biological Diversity (UNCBD). It is a periodic report that summarizes the latest data on the status and trends of biodiversity and draws conclusions relevant to the further implementation of the Convention.
The fourth edition of the Global Biodiversity Outlook was officially launched on the opening day of the Twelfth Meeting of the Conference of the Parties to the Convention on Biological Diversity (COP 12) in Pyeongchang, Korea. The report draws on various sources of information to provide a mid-term assessment of progress towards the implementation of the Strategic Plan for Biodiversity2011-2020.

70. Answer A
Difficulty Level: Easy
Explanation
The ‘Hargilla army’ is a group consisting of women from 14 self-help groups from Dadara, Pacharia and Singimari villages of Assam. Backed by the district administration and local conservation groups, the Hargilla Army has been successful in reducing threats and protecting habitat of the greater adjutant stork. Hargilla means "swallower of bones" in Sanskrit.The group won the United Nations Development Program India's 2016 Biodiversity Award.

71. Answer D
Difficulty Level: Medium
Explanation
With 132 member Governments, the Intergovernmental Science-Policy Platform on Biodiversity and Ecosystem Services (IPBES) is the global body that assesses the state of biodiversity and nature's contributions to people, in response to requests from decision-makers, and outlines options for the future based on different socio-economic choices.
Often described as the 'IPCC for Biodiversity', IPBES is the global science-policy forum tasked with providing the best available evidence to all decision-makers for people and nature.
The mission of IPBES is to strengthen policy and decisions through science, for the conservation and sustainable use of biodiversity, long-term human well-being and sustainable development.

72. Answer: b
Difficulty level: Medium
Explanation
• Sher Shah waged extensive wars with the Rajputs and expanded his empire.
• His conquests include Punjab, Malwa, Sind, Multan and Bundelkhand. His empire consisted of the whole of North India except Assam, Nepal, Kashmir and Gujarat.
•Sher Shah patronized the learned men. Malik Muhammad Jayasi wrote the famous Hindi work Padmavat during his reign.        


73. Answer : a
Launching the new logo of Steel Ministry “ISPATI IRADA”, Union Minister for Steel said that we must all work with an “Ispati Irada” to increase appropriate usage of steel in the country and bring more strength to society.

74. Answer : c

75. Answer : a
  • United Nations has declared 23rd September as International Sign Language Day.
  • This day was formally accepted by the United Nations Assembly on 19th December, 2017.
  • The theme of this year is “Sign Language-Rights for all”.
  • The objective of celebrating the sign language day is enhancing the awareness of sign language and making the reach of sign language to everyone.

76. Answer : b
  • The Union Minister for Science & Technology has recently launched UMMID (Unique Methods of Management and treatment of Inherited Disorders) initiative and inaugurated NIDAN (National Inherited Diseases Administration) Kendras, which is being supported by Department of Biotechnology (DBT), M/o Science and Technology.
  • UMMID initiative aims (i) to establish NIDAN Kendras to provide counselling, prenatal testing and diagnosis, management, and multidisciplinary care in Government Hospitals wherein the influx of patients is more, (ii) to produce skilled clinicians in Human Genetics, and (iii) to undertake screening of pregnant women and new born babies for inherited genetic diseases in hospitals at aspirational districts.

77. Answer : a

78. Answer : d
·         The Gujarat government has recently launched what is being described as the world’s first market for trading in particulate matter emissions.
·         While trading mechanisms for pollution control do exist in many parts of the world, none of them is for particulate matter emissions.
·         For example, the CDM (carbon development mechanism) under the Kyoto Protocol allows trade in ‘carbon credits’; the European Union’s Emission Trading System is for greenhouse gas emission; and India has a scheme run by the Bureau of Energy Efficiency that enables trading in energy units.
·         Launched in Surat, the Emissions Trading Scheme (ETS) is a regulatory tool that is aimed at reducing the pollution load in an area and at the same time minimising the cost of compliance for the industry.
·         ETS is a market in which the traded commodity is particulate matter emissions.
·         The Gujarat Pollution Control Board (GPCB) sets a cap on the total emission load from all industries.
·         Various industries can buy and sell the ability to emit particulate matter, by trading permits (in kilograms) under this cap.
·         For this reason, ETS is also called a cap-and-trade market.

79. Answer : c
  • PGS is a process of certifying organic products, which ensures that their production takes place in accordance with laid-down quality standards.
  • The certification is in the form of a documented logo or a statement.
  • The government’s 2015 PGS manual underlines that the system in India is based on “participatory approach, a shared vision, transparency and trust”.

80. Answer : d
  • A team of scientists from the University of Delhi and Zoological Survey of India (ZSI) discovered a new species of frog in Arunachal Pradesh and named it Microhyla eos, according to a statement.
  • Microhyla are a group of narrow-mouthed frogs (subfamily Microhylinae) that is primarily and widely distributed in Asia. 
  • Commonly known as 'Rice Frogs' or 'Chorus Frogs', the genus currently comprises of 49 recognised species. 
  • The new frog was discovered from riparian habitats in a primary evergreen forest in the Namdapha Tiger Reserve of the state, which is also the eastern-most protected area in the country.

81. Answer : c

82. Answer : c

83. Answer : a
  • It comes under the Ministry of Youth Affairs and Sports.

84.  Answer : d
  • India International Science Festival (IISF) was launched in 2015.
  • The aim is to engage the public with science and celebrate the joy of science and show the ways how science, technology, engineering and mathematics (STEM) provide us with the solutions to improve our lives. 
  • The 5th edition of India International Science Festival (IISF) 2019 will be held at Kolkata from 5th to 8th November, 2019.
  • The theme for this year’s festival is RISEN India – Research, Innovation, and Science Empowering the Nation.
  • Ministry of Science and Technology and Ministry of Earth Sciences in association with Vijnana Bharati (VIBHA), has created a unique platform of India International Science Festival which intends to inspire curiosity and make learning more rewarding.

85. Answer : c
The Portal will be available in all Indian languages within 90 days.
Unique Features
  1. Single window system for applying to any state or UT
  2. Integrated with Interoperable Criminal Justice System
  3. Payment Gateways for licensing fees
  4. E-signature
  5. Geo Tagging of office

86. Answer : d
  • Prime Minister Narendra Modi received the 2019 Global Goalkeeper Award for Swachh Bharat Mission from Bill and Melinda Gates Foundation recently.

87. Answer : d
  • Ministry of Agriculture has recently launched two mobile Apps – CHC Farm Machinery and Krishi Kisan App for Geo Tagging.
  • Through CHC Farm Machinery App, farmers can select and order the required machinery at the rates feasible for them from the Custom Hiring Centers located in the radius of 50 Kms. 
  • It is a Multi-lingual Mobile App.
  • Through this app, farmers, especially small and marginal farmers, will have easy access to high value and technical agricultural equipments which will facilitate optimum use of all types of inputs using these farming machines. 

88. Answer : b
  • Krishi Kisan App will provide farmers the information of best demonstration of high-yielding crops and seeds in their nearby area.
  • Any farmer with high quality of crops can utilise this platform to demonstrate best practices of cultivation to other farmers so that this will help other farmers also to adopt these methods.
  • The App will also help in geo-tagging and geo-fencing of crop and give weather forecast message to farmers. 

89. Answer : d
  • Under the Senior Citizens Savings Scheme (SCSS), a senior citizen can deposit up to Rs 15 lakh and the current interest rate is 8.6 per cent.
  • The tenure of this scheme is five years with the option to extend it for three more years.
  • However, the interest on SCSS is fully taxable, which is a major drawback of this scheme.
  • A study conducted recently by SBI has recommended that the government should consider exempting interest earned by senior citizens under SCSS from income tax.

90. Answer : d
  • Asperger syndrome is a developmental disorder.
  • Those who have it, find it difficult to communicate and interact.
  • The syndrome is also characterised with constricted and repetitive patterns.
  • Asperger’s syndrome is now considered to be a part of a wider category called autism spectrum disorder (ASD).
  • According to WebMD, those who have this syndrome do not generally make eye contact and might seem awkward during social interactions.
  • Exhibiting minimum emotions also counts as a symptom.
  • They also tend to talk about a particular topic with a lot of intensity.

91. Answer : a

92. Answer : c
  • The Financial Action Task Force (FATF) was established in July 1989 by a Group of Seven (G-7) Summit in Paris, initially to examine and develop measures to combat money laundering.
  • The objectives of the FATF are to set standards and promote effective implementation of legal, regulatory and operational measures for combating money laundering, terrorist financing and other related threats to the integrity of the international financial system.

93. A.   Initiative to quit tobacco by utilising mobile technology
Explanation :
Since its inception in September, 2018, the Union government’s tobacco Quitline, for counselling in south Indian languages, has received more than 5 lakh calls. The helpline 1800-11-2356 — started by the Union Health Ministry in April last year — is displayed on all tobacco products. The toll free Tobacco Quit-line Services [1800-11-2356] provides counseling and strategies for quitting tobacco use. Subsequently, the south Indian regional languages cell, NIMHANS Tobacco Quitline was started on September 11. It is monitored by National Institute of Mental Health and Neurosciences (NIMHANS) in Bengaluru.

94. C.   Both 1 and 2
Explanation :

An internal working group, formed by the RBI to review the current liquidity management framework with a view to simplifying it has submitted its report.
Recommendations made:
·         Guiding principles for an effective liquidity management framework should be: the liquidity framework should be guided by the objective of maintaining the call money rate close to the policy rate; it should be consistent with the policy rate; and it should not undermine the price discovery in the inter-bank money market.
·         The current liquidity management framework should largely continue in its present form- a corridor system with the call money rate as the target rate.

95. A.   It vests power in the RBI to give directions to banks and can take action.
Explanation :
The Reserve Bank of India (RBI) has placed several restrictions on Mumbai-based Punjab and Maharashtra Cooperative (PMC) Bank for six months. The bank cannot issue loans or open any fixed deposit accounts until February 2020. RBI has imposed the directions under sub-section (1) of Section 35A read with Section 56 of the Banking Regulation Act, 1949.
The Banking Regulation Act legislated in the year 1949 comprises a set of rules which govern the banking sector in India. The act vests powers in the RBI to grant licenses to banks as well as work as a banking regulator in India. Section 35A of the Banking Regulation Act, 1949 vests power in the RBI to give directions to banks and can take action, "to prevent the affairs of any banking company being conducted in a manner detrimental to the interests of the depositors or in a manner prejudicial to the interests of the banking company". The RBI under the act can also impose restrictions on banks to ensure better governance and control. Meanwhile, Section 56 of the act is applicable to cooperative societies. Hence, option (a) is the correct answer



96. D.   None of the above
Explanation :
Joint Military Exercise KAZIND-2019 between India and Kazakhstan will be conducted at Pithoragarh, Uttarakhand from 02 to 15 October 2019. Exercise KAZIND-2019 is the fourth edition of an annual event which is conducted alternatively in Kazakhstan and India. The exercise will comprise of nearly 100 soldiers from both Indian and Kazakhstan Army who would be sharing their experience gained during conduct of various counter insurgency and counter terrorism operations in the past. The aim of this exercise is to enhance the level of defence cooperation further fostering bilateral relations between the two nations. Hence, option (d) is the correct answer.

97. B.   2 only
Explanation :

Statement 1 is incorrect:
The Department of Drinking Water and Sanitation (DDWS), Ministry of Jal Shakti, GoI launched the 10 Year Rural Sanitation Strategy (2019-2029). This strategy has been prepared by Department of Drinking Water and Sanitation (DDWS), Ministry of Jal Shakti, in consultation with State Governments and other stakeholders.

98. C.   Both 1 and 2
Explanation :
Vice President of India Venkaiah Naidu presented the National Tourism Awards 2017-18 on World Tourism Day in New Delhi. World Tourism Day is celebrated annually on 27th September, to coincide with the anniversary of adoption of the statutes of the United Nations World Tourism Organisation (UNWTO). The theme for World Tourism Day 2019 is ‘Tourism and Jobs: a better future for all’. The UNWTO has selected India as the host country for official World Tourism Day 2019 celebrations. Hence both statements are correct.


99. A.   Periodical review of the performance of government servants with a view to ascertain whether they should be retained in service or retired in public interest.
Explanation :
The Central Board of Direct Taxes (CBDT) has compulsorily retired 15 more senior tax officers on corruption and other charges under Fundamental Rule 56(J) of the Central Civil Services (Pension) Rules. Since June, this is the fourth round of sacking of corrupt tax officials. In the previous three rounds, 49 high ranking tax officers, including 12 from the CBDT, were compulsorily retired. The action was in line with PM Modi's address to the nation from the Red Fort when he had said some black sheep in the tax administration may have misused their powers and harassed taxpayers, either by targeting honest assesses or taking excessive action for minor or procedural violations. 

100. B.                2 only
Explanation :

The SASTRA Ramanujan prize for 2019 will be awarded to mathematician Adam Harper, Assistant Professor with the University of Warwick, England for several outstanding contributions to analytic and probabilistic number theory.
The prize was instituted in 2005. Every year, this prize is awarded by SASTRA University on its campus near Kumbakonam in Tamil Nadu, on Ramanujan’s birth anniversary, December 22. The prize is conferred annually on mathematicians from across the world who are less than 32 years of age, working in an area influenced by the Srinivasa Ramanujan. The age limit is 32 years to commemorate the fact that Ramanujan accomplished a phenomenal body of work in this short span. It carries a citation and an award of $10,000. The SASTRA-Ramanujan Award has gained global repute ever since it was instituted as four mathematicians, including Manjul Bhargava and Akshay Venkatesh, who were awarded this prize have gone on to win the Fields Medal later. Hence only statement 2 is correct.



1 comment: